Mostrar Mensajes

Esta sección te permite ver todos los posts escritos por este usuario. Ten en cuenta que sólo puedes ver los posts escritos en zonas a las que tienes acceso en este momento.

Mensajes - chien-chat

Páginas: [1] 2 3
1
Gracias, me ha quedado claro.
Saludos.

2
Sean \( X,Y \) dos espacios vectoriales, \( A\subset X \). Considere la aplicación lineal \( T:X\rightarrow Y \). Pruebe que \( T^{-1}(T(A))=T^{-1}(0)+A \).

Hola, he intentado una inclusión:

Se tiene \( A\subset T^{-1}(T(A)) \).
Sea \( u\in T^{-1}(T(A)) \Rightarrow T(u)\in T(A) \Rightarrow T^{-1}(T(u))\in A \subset
A+T^{-1}(0) \).
Por tanto \( T^{-1}(T(A))\subset T^{-1}(0)+A \).

Si mi prueba es correcta, podrían ayudarme a demostrar la otra inclusión?
Saludos.

3
Geometría Diferencial - Variedades / Curva parametrizada
« en: 23 Noviembre, 2018, 12:16 am »
Una curva parametrizada \( a(t) \) tiene la propiedad que su segunda derivada \( a''(t) \) es identicamente cero. ¿que puede decir de \( a \)?

Int: Sea \( a(t) = (a_1(t),a_2(t),a_3(t))\in \mathbb{R}^3 \Rightarrow a''(t)=(a_1''(t),a_2''(t),a_3''(t)) \Rightarrow a'' \equiv 0 \Leftrightarrow a_i'' = 0; i=1,2,3 \Rightarrow a_i = m_it+n_i; i=1,2,3 \). Es decir, la curva \( a \) es la ecuación de una recta.  ???

4
Matemática Discreta y Algoritmos / Re: Factorización en Fq[x]
« en: 03 Septiembre, 2018, 09:02 pm »
Hola. Respondiendo a tu pregunta, es si, pues es la idea del curso ahora.
Anteriormente aprendimos a factorizar ''normalmente'' por así decirlo, es decir, buscar los ceros (los cuáles en mi problema son 2 y 3) y luego factorizar usando la idea de Ruffini, pero en mod 5. No sé si me explico.

Saludos cordiales ;D

5
Matemática Discreta y Algoritmos / Factorización en Fq[x]
« en: 03 Septiembre, 2018, 06:51 pm »
Hola, estoy teniendo problemas al factorizar el siguiente polinomio en \( \mathbb{F}_5\left [ x \right ] \), alguien podria orientarme por favor :'(
\( x^5+2x^4+3x^3+x^2+2x+4 \in \mathbb{F}_5\left [ x  \right ] \).

He aquí mi intento de lo que he podido entender: La idea es usar el algoritmo de Cantor&Zasenhauss.

Sea \( f(x) := x^5+2x^4+3x^3+x^2+2x+4 \in \mathbb{F}_5\left [ x  \right ] \Rightarrow f'(x) = 3x^3+4x^2+2x+2 \in \mathbb{F}_5\left [ x  \right ] \). Luego, comprobamos que \( f(x) \) sea libre de cuadrados, ie que \( \gcd(f(x),f'(x))=1 \), lo cual no se verifica, ya que \( \gcd(f(x),f'(x))=x+2 \), por ende, hacemos \( g(x) := \dfrac{f(x)}{\gcd(f(x),f'(x))}=x^4+3x^2+2 \Rightarrow g'(x)=4x^3+x \) y así obtenemos que \( \gcd(g(x),g'(x))=1 \), ie \( g(x) \) es libre de cuadrados.

De aquí me empiezo a complicar: (Para los sgtes pasos me estoy guiando de acá http://planetmath.org/cantorzassenhaussplit)
Usaré las mismas notaciones de la página anexa:

\( B_1 = A, \ B_{k+1} := \displaystyle\frac{A}{\gcd(B_k, x^{5^k}-x)}   \)

Entonces, tenemos:

\( B_1 := x^4+3x^2+2  \)
\( B_2 := \displaystyle\frac{x^4+3x^2+2}{\gcd(B_k, x^5-x)}=x^3+2x^2+2x+4 \)
\( B_3 := \displaystyle\frac{x^4+3x^2+2}{\gcd(B_k, x^{25}-x)}=x^2+1 \)

Y luego cómo prosigo?? alguien puede ayudarme por favor
de antemano gracias
saludoss  :laugh:

6
Ecuaciones diferenciales / Sistema hiperbólico
« en: 21 Agosto, 2018, 05:34 pm »
Sea \( \mathcal{M}_n \) el conjunto de todas las matrices \( n\times n \) identificadas con \( \mathbb{R}^{n^2} \) y \( S:=\lbrace A\in \mathcal{M}_n : x′=Ax \text{ es hiperb[b]ó[/b]lico}\rbrace. \) Muestre que \( S \) es abierto y denso en \( \mathcal{M}_n \).

¿Alguien puede ayudarme con este problema, por favor?  :'( El ejercicio es extracto del libro: Lições de equações diferenciais ordinárias - Jorge Sotomayor: exercise 28, page 101.
De antemano muchas gracias.

7
Topología (general) / Re: Espacio metrizable
« en: 13 Agosto, 2018, 10:34 pm »
Gracias Eparoh por contestar y por la ayuda; había entendido lo anterior dicho y por eso borré parte del mensaje.
Aún así, con tu respuesta me quedó más claro aún.

Saludos.

8
Topología (general) / Re: Espacio metrizable
« en: 13 Agosto, 2018, 10:09 pm »
Gracias por responder. Me quedó más claro.

9
Topología (general) / Espacio metrizable
« en: 13 Agosto, 2018, 05:56 pm »
Sean \( X,Y \) espacios topológicos. Let \( f:X\rightarrow Y \) función tal que para cada sucesión convergente \( x_n\rightarrow x \) en \( X \), \( (f(x_n)) \) converge a \( f(x) \). Demuestre que si el espacio \( X \) es metrizable \( \Rightarrow f \) es continua.

10
Matemática Discreta y Algoritmos / Regla de Cramer con magma
« en: 28 Junio, 2018, 10:52 am »
Hola.

Estoy estudiando aplicaciones de la aritmética modular, en particular sistemas lineales sobre los racionales.
 
Tengo un lío con este problema: La idea es que, con ayuda de MAGMA calculator, se resuelvan sistemas lineales sobre los racionales \( \mathbb{Q} \) con la regla de Cramer y el método de computación modular (más bien diría yo entender el proceso que hace la calculadora magma).

El primer ejercicio que se da es para una matriz \( A\in \mathcal{M}_{7\times 7}(\mathbb{Q}) \) y un vector \( b\in \mathbb{Q}^7 \) para el cuál se pide encontrar lo siguiente:
1) un sistema equivalente con coeficientes enteros.   (LISTO) \( \checkmark \)
2) una cota para valores absolutos de numeradores y denominadores reducidos de cada \( x_i \).   (LISTO) \( \checkmark \)
3) un conjunto de números primos \( p_j \) para resolver el sistema\( \mod p_j \).

Con este último ítem tengo problemas. Como ahora el nuevo sistema matriz-vector se trabaja en enteros, tenemos \( A'\in \mathcal{M}_{7\times 7}(\mathbb{Z}) \) y \( b'\in \mathbb{Z}^7 \), entonces ¿cuáles números primos elijo?.
Mi idea es encontrar \( p_1,p_2,...,p_n \) enteros positivos tales que \( gcd(p_i,p_j)=1 \) para \( i\neq j \). Luego, tenemos los números enteros \( b_1,b_2,...,b_7 \) del vector \( b' \), por lo que el sistema formado por las congruencias \( x_i\equiv b_1(\mod m_1) \), \( x_i\equiv b_2(\mod m_2) \),...,\( x_i\equiv b_7(\mod p_n) \) (\( i=1,...7 \)) tiene sol. única salvo \( p=p_1p_2...p_n \) (Esto último que escribí lo saqué de un corolario en internet --- Teo. chino de los restos).

Estoy en confusión con este apartado 3, ¿alguién podría orientarme mejor? o ayudarme verlo de otra mejor manera para abordarlo mejor, porfavor, de antemano gracias.

Saludos cordiales.

11
Topología (general) / Re: Topología sobre e.v.
« en: 31 Marzo, 2018, 02:32 pm »
Fíjate que para todo \( x\in\bigcup_{\alpha\in I} A_\alpha \) se cumple que existe un \( r>0 \) tal que \( B(x,r)\subset\bigcup_{\alpha\in I} A_\alpha \), por tanto...
Por tanto, como cada \( A_\alpha \in \tau_d \Rightarrow \displaystyle \bigcup_\alpha A_\alpha \in \tau_d \)  ???
Citar
Por otro lado es en general falso que \( \bigcap_{\alpha\in I} A_\alpha\in\tau_d \). El axioma sobre intersecciones dice que sólo las intersecciones finitas entre abiertos pertenecen necesariamente a la topología. Por tanto si \( |I|\ge\aleph_0 \) entonces no se cumple necesariamente que \( \bigcap_{\alpha\in I} A_\alpha\in\tau_d \).
Verdad. Podría tomar entonces \( A_1,...,A_n \in \tau_d \). Se debe demostrar entonces que \( \displaystyle \bigcap_{i=1}^nA_i \in \tau_d \). Sea \( x\in \displaystyle \bigcap_{i=1}^nA_i \), entonces para cada \( i=1,...,n \) se tiene que \( x \) pertenece a cada \( A_i\in \tau_d \), por lo que \( \displaystyle \bigcap_{i=1}^nA_i \in \tau_d \). Es decir, \( \tau_d \) es una topología sobre el conjunto \( X \).

12
Topología (general) / Topología sobre e.v.
« en: 31 Marzo, 2018, 11:20 am »
Sea \( X \) un conjunto y \( d:X\times{X}\rightarrow{\mathbb{R}} \) una distancia en \( X \), donde \( (X,d) \) es un espacio métrico. Demuestre que \( \tau_d=\left\{A\subseteq{X}\ ;\  \forall{a\in A},\ \exists{r>0}\ \text{tal que}\ B(a,r)\subset{A}\right\}  \) es una topología sobre \( X \).

Hola, tengo un enredo con este problema. He intentado lo siguiente:
Notemos primero que \( \emptyset \in{\tau_d},\ X\in \tau_d \). Ahora, consideremos una familia \( \left\{{A_\alpha}\right\}_{\alpha \in I} \subset{\tau_d} \). Debemos demostrar que \( \displaystyle \bigcup_{\alpha}A_\alpha \in \tau_d \) y también que \( \displaystyle \bigcap_{\alpha}A_\alpha \in \tau_d \).
Sabemos que \( B(a,r)=\left\{{x\in X;\ d(a,x)<r}\right\} \). Como \( A_\alpha \in \tau_d \Rightarrow A_\alpha \subseteq{X}\ ;\  \forall a\in A_\alpha,\ \exists r>0 \ \text{tal que } B(a,r)\subset{A_\alpha} \), es decir, podemos considerar que para cada \( i=1,...,n \) se tiene \( A_i\in \tau_d \Longleftrightarrow A_i \subseteq{X};\ \forall a_i\in A_i,\ \exists r_i>0\ \text{tal que}\ B(a_i,r_i)\subset{A_i} \). Primero demostraremos que \( \displaystyle \bigcup_\alpha A_\alpha \in \tau_d \), donde \( \displaystyle \bigcup_{i=1}^nB(a_i,r_i)\subset \bigcup_{i=1}^nA_i \). Notemos que \( \displaystyle \bigcup_{i=1}^n B(a_i,r_i)=\bigcup_{i=1}^n \left\{{x\in X: d(a_i,x)<r_i}\right\} \).

Y aquí he quedado, no he logrado finalizar esa parte de la demostración.
Algún hint? ??? De antemano gracias

13
Geometría Diferencial - Variedades / Re: Integrales en variedades
« en: 21 Diciembre, 2017, 07:58 am »
Hola
Para no dejar respuestas en blanco, aquí mis soluciones, espero que estén bien si mis calculos no me fallan:

a) \( \displaystyle \int_M(x+y+z)dM \) para \( M=\left\{{(x,y,z)\in \mathbb{R}^3: x^2+y^2+z^2=a^2, z \geq 0}\right\} \).

Spoiler
Se tiene que \( M=\left \{ (x,y,z)\in \mathbb{R}^3:z=\sqrt{a^2-x^2-y^2};\ x^2+y^2\leq a^2 \right \} \) es una superficie compacta orientable con borde, entonces consideremos una carta \( h^{-1}:V\rightarrow M \) que cubre toda la variedad \( M \), definida por: \( h^{-1}(\theta,\phi)=a(cos(\theta) sen(\phi),sen(\theta) sen(\phi),cos(\phi)) \); \( a\in \mathbb{R};\ \theta \in [0,2\pi];\ \phi \in [0,\pi /2] \)
Usando los coeficientes de la métrica Riemanniana, se obtiene:
\begin{align*}
E(\theta,\phi)=g_{11}(\theta,\phi)=\left \langle \dfrac{\partial h^{-1}}{\partial \theta},\dfrac{\partial h^{-1}}{\partial \theta}  \right \rangle (\theta,\phi) &= \left \Vert \dfrac{\partial h^{-1}(\theta,\phi)}{\partial \theta} \right \Vert^2 \\
 &= \Vert a(-sen(\theta)sen(\phi),cos(\theta)sen(\phi),0)\Vert^2 \\
 &= a^2(sen^2(\theta)sen^2(\phi)+cos^2(\theta)sen^2(\phi))=a^2sen^2(\phi)\\
G(\theta,\phi)=g_{22}(\theta,\phi)=\left \langle \dfrac{\partial h^{-1}}{\partial \phi},\dfrac{\partial h^{-1}}{\partial \phi}  \right \rangle (\theta,\phi) &= \left \Vert \dfrac{\partial h^{-1}(\theta,\phi)}{\partial \phi} \right \Vert^2 \\
 &= \Vert a(cos(\theta)cos(\phi),sen(\theta)cos(\phi),-sen(\phi))\Vert^2 \\
 &= a^2(cos^2(\theta)cos^2(\phi)+sen^2(\theta)cos^2(\phi)+sen^2(\phi))=a^2\\
 F(\theta,\phi)=g_{12}(\theta,\phi) = \left \langle \dfrac{\partial h^{-1}}{\partial \theta},\dfrac{\partial h^{-1}}{\partial \phi}  \right \rangle (\theta,\phi) &= a^2(-sen(\theta)sen(\phi)cos(\theta)cos(\phi)+cos(\theta)sen(\phi)sen(\theta)cos(\phi))=0\\
\end{align*} \( \Rightarrow dM=\sqrt{g}\ d\theta \wedge d\phi=\sqrt{EG-F^2}d\theta \wedge d\phi=\sqrt{a^4sen^2(\phi)}d\theta \wedge d\phi = a^2sen(\phi)d\theta \wedge d\phi \)
Sea \( f:M\rightarrow \mathbb{R} \) una función defininda por \( f(x,y,z)=x+y+z \)
\( \Rightarrow \displaystyle \int_M fdM = \int_Mf(h^{-1})\sqrt{g}\ d\theta d\phi = \int_{0}^{\pi/2} \int_{0}^{2\pi}(acos(\theta)sen(\phi)+asen(\theta)sen(\phi)+acos(\phi))\ a^2sen(\phi)\ d\theta d\phi \)
\( = a^3 \displaystyle \int_{0}^{\pi/2}\int_{0}^{2\pi}(cos(\theta)sen^2(\phi)+sen(\theta)sen^2(\phi)+cos(\phi)sen(\phi))\ d\theta d\phi = a^3 \displaystyle \int_{0}^{\pi/2}((sen(\theta)-cos(\theta))sen^2(\phi)+\theta cos(\phi)sen(\phi))\mid _{\theta=0}^{\theta=2\pi}\ d\phi  \)
\( = 2\pi a^3 \displaystyle \int_{0}^{\pi/2}cos(\phi)sen(\phi)\ d\phi = \displaystyle 2\pi a^3 \int_{0}^{1}u\ du = 2\pi a^3 \left(\dfrac{u^2}{2} \right)_{u=0}^{u=1}=\pi a^3 \Rightarrow \displaystyle \int_M (x+y+z)\ dM=\pi a^3 \)
[cerrar]

b) \( \displaystyle \int_M(x^2+y^2)\ dM \) donde \( M \) es la frontera del subconjunto de \( \mathbb{R}^3 \) descrito por la desigualdad \( \sqrt{x^2+y^2}\leq z \leq 1 \).

Spoiler
Sea \( S=\lbrace (x,y,z)\in \mathbb{R}^3:\sqrt{x^2+y^2}\leq z\leq 1 \rbrace \), entonces la frontera \( M=\partial S \) es descrita por:
\( \partial S=\left \lbrace (x,y,z)\in \mathbb{R}^3:x^2+y^2=z^2;\ 0\leq z\leq 1 \right \rbrace \cup \left \lbrace (x,y,z)\in \mathbb{R}^3: x^2+y^2\leq 1,\ z=1 \right \rbrace \), por lo tanto, sean \( S_1=\left \lbrace (x,y,z)\in \mathbb{R}^3:x^2+y^2=z^2;\ 0\leq z\leq 1 \right \rbrace \) y \( S_2=\left \lbrace (x,y,z)\in \mathbb{R}^3: x^2+y^2\leq 1,\ z=1 \right \rbrace \), además consideremos dos cartas \( h_1^{-1}:V\rightarrow S_1 \) y \( h_2^{-2}:V\rightarrow S_2 \) definidas por
\( h_1^{-1}(r,\theta)=(rcos(\theta),rsen(\theta),r);\ r\in [0,1];\ \theta \in [0,2\pi] \)
\( h_2^{-1}(r,\theta)=(rcos(\theta),rsen(\theta),1);\ r\in [0,1];\ \theta \in [0,2\pi] \)

Sea \( f:M\rightarrow \mathbb{R} \) una función definida por \( f(x,y,z)=x^2+y^2 \).

Para el conjunto \( S_1 \) se tiene que:
\( E(r,\theta)=g_{11}(r,\theta)=\left \langle \dfrac{\partial h_1^{-1}}{\partial r},\dfrac{\partial h_1^{-1}}{\partial r} \right \rangle (r,\theta)=\Vert (cos(\theta),sen(\theta),1)\Vert^2=cos^2(\theta)+sen^2(\theta)+1=2 \)
\( G(r,\theta)=g_{22}(r,\theta)=\left \langle \dfrac{\partial h_1^{-1}}{\partial \theta},\dfrac{\partial h_1^{-1}}{\partial \theta} \right \rangle (r,\theta)=\Vert (-rsen(\theta),rcos(\theta),0)\Vert^2=r^2sen^2(\theta)+r^2cos^2(\theta)=r^2 \)
\( F(r,\theta)=g_{12}(r,\theta)=\left \langle \dfrac{\partial h_1^{-1}}{\partial r},\dfrac{\partial h_1^{-1}}{\partial \theta} \right \rangle (r,\theta)=\langle (cos(\theta),sen(\theta),1)\ , \ (-rsen(\theta),rcos(\theta),0) \rangle=0 \)
\( \Rightarrow dS_1=\sqrt{g}\ dr\wedge d\theta=\sqrt{EG-F^2}\ dr\wedge d\theta=\sqrt{2r^2}\ dr\wedge d\theta=r\sqrt{2}\ dr\wedge d\theta \)
\( \Rightarrow \displaystyle \int_{S_1}fdS_1=\int_{S_1}f(h_1^{-1})\sqrt{g}\ drd\theta=\int_0^{2\pi}\int_{0}^1(r^2cos^2(\theta)+r^2sen^2(\theta))r\sqrt{2}\ drd\theta=\sqrt{2}\int_0^{2\pi}\int_0^1r^3\ drd\theta \)
\( =\displaystyle 2\sqrt{2}\pi \int_0^1r^3\ dr=2\sqrt{2}\pi\left(\dfrac{r^4}{4} \right)_{r=0}^{r=1}=2\sqrt{2}\pi \left(\dfrac{1}{4} \right)=\dfrac{\pi \sqrt{2}}{2} \)

Para el conjunto \( S_2 \) se tiene que:
\( E(r,\theta)=g_{11}(r,\theta)=\left \langle \dfrac{\partial h_2^{-1}}{\partial r},\dfrac{\partial h_2^{-1}}{\partial r} \right \rangle (r,\theta)=\Vert (cos(\theta),sen(\theta),0)\Vert^2=cos^2(\theta)+sen^2(\theta)=1 \)
\( G(r,\theta)=g_{22}(r,\theta)=\left \langle \dfrac{\partial h_2^{-1}}{\partial \theta},\dfrac{\partial h_2^{-1}}{\partial \theta} \right \rangle (r,\theta)=\Vert (-rsen(\theta),rcos(\theta),0)\Vert^2=r^2sen^2(\theta)+r^2cos^2(\theta)=r^2 \)
\( F(r,\theta)=g_{12}(r,\theta)=\left \langle \dfrac{\partial h_2^{-1}}{\partial r},\dfrac{\partial h_2^{-1}}{\partial \theta} \right \rangle (r,\theta)=\langle (cos(\theta),sen(\theta),0)\ , \ (-rsen(\theta),rcos(\theta),0) \rangle=0 \)
\( \Rightarrow dS_2=\sqrt{g}\ dr\wedge d\theta=\sqrt{EG-F^2}\ dr\wedge d\theta=r\ dr\wedge d\theta \)
\( \Rightarrow \displaystyle \int_{S_2}fdS_2=\int_{S_2}f(h_2^{-1})\sqrt{g}\ drd\theta=\int_0^{2\pi}\int_{0}^1(r^2cos^2(\theta)+r^2sen^2(\theta))r\ drd\theta=\int_0^{2\pi}\int_0^1r^3\ drd\theta \)
\( =2\pi \left(\dfrac{r^4}{4}\right)_{r=0}^{r=1}=2\pi \left(\dfrac{1}{4}\right)=\dfrac{\pi}{2} \)
Por lo tanto \( \displaystyle \int_MfdM=\int_{S_1}fdS_1+\int_{S_2}fdS_2=\dfrac{\pi \sqrt{2}}{2}+\dfrac{\pi}{2}=\left(\dfrac{\sqrt{2}+1}{2}\right)\pi \Rightarrow \int_M(x^2+y^2)dM=\left(\dfrac{\sqrt{2}+1}{2}\right)\pi \)
[cerrar]

Saludos.

14
Geometría Diferencial - Variedades / Ecuación del calor
« en: 03 Diciembre, 2017, 08:17 pm »
Sea \( M\subset \mathbb{R}^n \) una variedad compacta y orientada, y asuma que \( f:M\times{[0,\infty)} \rightarrow{\mathbb{R}} \) es suave. La ecuación de calor es \( \Delta_x f(x,t)=\dfrac{\partial f(x,t)}{\partial t} \). Pruebe que si \( f \) es una solución de la ecuación de calor que satisface \( f(x,0)=0, \forall{x\in M} \) y \( f(y,t)=0, \forall{y\in \partial M}, \forall{t\in [0,t_0]} \), entonces \( f\equiv{0} \) en el conjunto \( M\times [0,t_0] \).

Hola, alguien podría darme una mano con este problema?
He intentado lo siguiente: \( f(x,t)=\displaystyle \int_{M\times [0,\infty)} \Delta_x f(x,t) \partial t = \int_{M\times [0,\infty)} \dfrac{\partial^2 f(x,t)}{\partial x^2} \partial t =
 \int_{M\times [0,\infty)} div_x(grad_x (f(x,t))) \partial t \)   ???

De aquí me complico, no se me ocurre una idea de como seguir  :'(
Saludos y gracias deantemano

15
Geometría Diferencial - Variedades / Re: Subvariedad compacta
« en: 02 Diciembre, 2017, 09:19 pm »
Gracias Luis, he intentado lo siguiente:

Teo: Sea \( M \) variedad compacta y orientada y sea \( N \) el campo de vectores normal exterior unitario a su frontera.
Entonces para todo campo de vectores \( X\in \mathcal{X}(M) \) se tiene \( \displaystyle \int_M div(X)dM = \int_{\partial M}\left<{X,N}\right>d(\partial M) \)

Sea \( T \) subvariedad compacta \( n- \)dimensional de \( \mathbb{R}^n \) y \( V\in \mathbb{R}^n \) un vector constante. Entonces:
\( \displaystyle \int_{M^{n-1}} \left<{N(x),V}\right>dM^{n-1} = \int_{\partial T} \left<{N(x),V}\right>d(\partial T) = \int_T div(V) dT = \int_T 0 dT = 0  \)

16
Geometría Diferencial - Variedades / Subvariedad compacta
« en: 30 Noviembre, 2017, 12:25 am »
Sea \( M^{n-1} \) la frontera de una subvariedad compacta \( n- \)dimensional de \( \mathbb{R}^n \) y \( N(x) \) el campo de vectores normal unitario (exterior).
Si \( V\in \mathbb{R}^n \) es un vector constante, entonces \( \displaystyle \int_{M^{n-1}} \left<{N(x),V}\right>dM^{n-1} = 0 \)

Hola, estoy algo complicada con este ejercicio, espero puedan ayudarme.
Verán, sea \( V = (v_1,...,v_n) \in \mathbb{R}^n \) el vector constante, \( \displaystyle X = (x_1,...,x_n) \in \mathbb{R}^n \Rightarrow N(X)=x_1 \frac{{\partial}}{{\partial x_1}}+...+x_n\frac{{\partial}}{{\partial x_n}} \) será el campo vectorial exterior normal unitario.
Aquí es donde me complico, en el producto interior:
Spoiler
No estoy segura de esto  :'(: \( \displaystyle \left<{N(x),V}\right> = \left< \left ( x_1 \frac{{\partial}}{{\partial x_1}}+...+x_n\frac{{\partial}}{{\partial x_n}} \right ) ,(v_1,...,v_n)\right> = x_1\frac{\partial v_1}{\partial x_1}+...+x_n\frac{\partial v_n}{\partial x_n} = x_1*0+...+x_n*0=0 \), entonces \( \displaystyle \int_{M^{n-1}} \left<{N(x),V}\right>dM^{n-1} = 0 \)  ???
[cerrar]

Saludos

17
Geometría Diferencial - Variedades / Re: Integrales en variedades
« en: 27 Noviembre, 2017, 12:46 am »
Si, fué error de tipeo, gracias por la ayuda, pude lograr hacerlo  :aplauso:

Para la parte b) al integrar en los dos trozos, la integral resultante pedida sería sumar ambas contribuciones?  ::)

18
Geometría Diferencial - Variedades / Integrales en variedades
« en: 22 Noviembre, 2017, 08:25 am »
Calcule las siguientes integrales de superficie:
a) \( \displaystyle \int_M(x+y+z)\ dM \) para \( M=\left\{{(x,y,z)\in \mathbb{R}^3: x^2+y^2+z^2=a^2, z \geq 0}\right\} \)
b) \( \displaystyle \int_M (x^2+y^2)\ dM \), donde \( M \) is the boundary del subconjunto de \( \mathbb{R}^3 \) descrito por la desigualdad \( \sqrt{x^2+y^2}\leq z \leq 1 \).

Para la a) he hecho lo siguiente: corregirme por favor :(
Hay que integrar en variedades.

\( M=\left\{{(x,y,z)\in \mathbb{R}^3: z=\sqrt{a^2-x^2-y^2}\geq 0, x^2+y^2\leq a^2}\right\} \) es una superficie compacta orientable con borde.
Si \( h:V\subset{\mathbb{R}^2} \rightarrow M \) es una parametrización de \( M \), entonces, usando los coeficientes de la métrica Riemanniana, se tiene que \( \sqrt{g} = a^2 cos(\phi)  \), donde \( h(a,\theta,\phi)=(asin(\theta)cos(\phi),asin(\theta)sin(\phi),acos(\phi)); a>0, \theta \in [0,2\pi], \phi \in [0, \pi /2]  \)

Acá no estoy muy segura de mi procedimiento:
Sea \( f:M\rightarrow{\mathbb{R}} \) definida por \( f(x,y,z)=x+y+z \Rightarrow \displaystyle \int_MfdM=\int f(h(r,\theta,\phi))\sqrt{g(r,\theta,\phi)}drd\theta d\phi; r\in [0,a], \theta \in [0, 2\pi], \phi \in [0, \pi /2] \)
En donde esta última integral es 0.  ??? ???
Para la b) podrían darme algún hint?

gracias de antemano
Saludos,

19
Geometría Diferencial - Variedades / n-forma exterior dV
« en: 31 Agosto, 2017, 10:32 pm »
Demuestre que \( dV\in \Lambda^n(V^*) \) y que su definición es independiente de la base ortonormal positiva que se considere.

Idea: Sea \( (V,g) \) espacio vectorial sobre \( \mathbb{R} \) con un producto escalar no degenerado \( g \), y \( e_1,...,e_n \) base ortonormal de \( V \) con respecto a \( g \) positivamente orientada.
Sea \( dV: \underset{n-veces}{\underbrace{V\times ... \times V}} \rightarrow \mathbb{R} \) la aplicación definida por \( dV(v_1,...,v_n)=det \begin{pmatrix}
g(v_1,e_1) & ... & g(v_n,e_1)\\
\vdots & \ddots & \vdots\\
g(v_1,e_n) & ... & g(v_n,e_n)
\end{pmatrix} \Rightarrow dV \in \Lambda^n(V^*) \)
Ahora, el lío es el siguiente ¿porque es independiente de la base ortonormal positiva que se considere? :o Saludos...

20
Sea \( \omega \in \Lambda^k(V^*) \) y \( v_o\in V \). Pruebe que \( i_{v_0}(\omega)\in \Lambda^{k-1}(V^*) \).

\( i_{v_0}(\omega) \) se llama el producto interior del vector \( v_0 \) con la \( k- \)forma exterior \( \omega \).
A ver, por la definición supongo debiera bastar.. es decir, si dados \( \omega \in \Lambda^k(V^*) \) y \( u_0 \in V \) se define
\( i_{u_0}(\omega):\underset{(k-1)-veces}{\underbrace{V\times ... \times V}} \rightarrow \mathbb{K} \) por \( i_{u_0}(\omega)(v_1,...,v_{k-1})=\omega(u_0,v_1,...,v_{k-1}) \Rightarrow i_{u_0}(\omega) \in \Lambda^{k-1}(V^*) \), luego si \( u_0=v_0 \) estamos.   ???

Páginas: [1] 2 3